What is the largest number of mutually obtuse vectors in Rn?

Click For Summary
SUMMARY

The largest number of mutually obtuse vectors in ℝn is determined by the dimensionality of the space, specifically that for any set of m vectors, if m = n + 1, the vectors must be linearly dependent. This conclusion is supported by the argument that in an n-dimensional space, having more than n vectors leads to linear dependence, which contradicts the requirement of pairwise orthogonality (vi⋅vj = 0 for all i ≠ j). The discussion references solutions found in external resources, including a MathOverflow post and a Stanford homework problem, which clarify the conditions under which contradictions arise.

PREREQUISITES
  • Linear independence in vector spaces
  • Understanding of orthogonality and dot products
  • Familiarity with n-dimensional spaces
  • Basic knowledge of inner product spaces
NEXT STEPS
  • Study the concept of linear dependence in n-dimensional vector spaces
  • Explore the properties of inner product spaces and their implications
  • Research the Kissing number problem and its relation to vector arrangements
  • Learn about the implications of dimensionality on vector sets in linear algebra
USEFUL FOR

Students of linear algebra, mathematicians exploring vector spaces, and anyone interested in the properties of orthogonal vectors in higher dimensions.

RandomGuy1
Messages
19
Reaction score
0
This is my question:

What is the largest m such that there exist v1, ... ,vm ∈ ℝn such that for all i and j, if 1 ≤ i < j ≤ m, then ≤ vivj = 0

I found a couple of solutions online.
http://mathoverflow.net/questions/31436/largest-number-of-vectors-with-pairwise-negative-dot-product
http://math.stanford.edu/~akshay/math113/hw7.pdf (problem 10. But it's basically the same solution as the one in the link above)

I kind of understand the contradiction but I don't get why there won't be a contradiction when you take m = n + 1. This is my first course in linear algebra, so far I've learned about linear independence, subspaces, orthogonality but I'm not very familiar with things like inner product spaces - only dot products. I need someone to explain this in simpler terms.
 
Physics news on Phys.org
RandomGuy1 said:
This is my question:

What is the largest m such that there exist v1, ... ,vm ∈ ℝn such that for all i and j, if 1 ≤ i < j ≤ m, then ≤ vivj = 0

I found a couple of solutions online.
http://mathoverflow.net/questions/31436/largest-number-of-vectors-with-pairwise-negative-dot-product
http://math.stanford.edu/~akshay/math113/hw7.pdf (problem 10. But it's basically the same solution as the one in the link above)

I kind of understand the contradiction but I don't get why there won't be a contradiction when you take m = n + 1. This is my first course in linear algebra, so far I've learned about linear independence, subspaces, orthogonality but I'm not very familiar with things like inner product spaces - only dot products. I need someone to explain this in simpler terms.
I'm referring to the argument in the pdf.

In case b), where M or N is 0, they use the last, until then unused, vector, ##v_{n+2}##, to get the contradiction. So we need one vector kept on the side.

Now let's go back to the start of the proof. If m=n+1, and we have to keep one vector on the side, there are only n vectors left to work with. But the argument started with noticing that the n+1 vectors must be linearly dependent. That worked because we had n+1 vectors in an n-dimensional space. With n vectors in an n-dimensional space, you can't be sure that they are linearly dependent. Hence, you don't have the equation marked by (*) in the pdf.
 
  • Like
Likes RandomGuy1
I got it. Thank you so much.
 
It is interesting that the question is quite easy to answer for 90 degrees, but an unsolved problem (Kissing number problem) for 60 degrees.
 
as often happens your problem is incorrectly stated here. you have to specify non zero vectors to get a contradiction. i.e. a sequence of zero vectors, no matter how long, satisfies the statement you gave above.
 
I am studying the mathematical formalism behind non-commutative geometry approach to quantum gravity. I was reading about Hopf algebras and their Drinfeld twist with a specific example of the Moyal-Weyl twist defined as F=exp(-iλ/2θ^(μν)∂_μ⊗∂_ν) where λ is a constant parametar and θ antisymmetric constant tensor. {∂_μ} is the basis of the tangent vector space over the underlying spacetime Now, from my understanding the enveloping algebra which appears in the definition of the Hopf algebra...

Similar threads

  • · Replies 4 ·
Replies
4
Views
3K
  • · Replies 19 ·
Replies
19
Views
4K
Replies
15
Views
2K
  • · Replies 8 ·
Replies
8
Views
4K
Replies
4
Views
3K
Replies
8
Views
4K
Replies
3
Views
6K
  • · Replies 15 ·
Replies
15
Views
3K
Replies
4
Views
5K
  • · Replies 1 ·
Replies
1
Views
4K